LSAT and Law School Admissions Forum

Get expert LSAT preparation and law school admissions advice from PowerScore Test Preparation.

User avatar
 Dave Killoran
PowerScore Staff
  • PowerScore Staff
  • Posts: 5853
  • Joined: Mar 25, 2011
|
#45667
Complete Question Explanation
(The complete setup for this game can be found here: lsat/viewtopic.php?t=1064)

The correct answer choice is (C)

If L is the only workshop on Monday (Template #2B in the setup) then S must be on Tuesday and Wednesday. That means that one of either P or R could be scheduled on Wednesday with S, or P and R could both be scheduled for Thursday and Friday. Since it could be true that S is the only workshop scheduled for Wednesday, answer choice (C) is correct.
 mattnj
  • Posts: 13
  • Joined: Aug 19, 2017
|
#41644
Shouldn't these 2 combinations be valid as well:
M-T-W-Th-F
L-LS-SP-PR-R or L-LS-SR-RP-P

Or did I miss a rule or inference?

If they are valid, how does it impact the answer to question 12?

Thank you!
 James Finch
PowerScore Staff
  • PowerScore Staff
  • Posts: 943
  • Joined: Sep 06, 2017
|
#41675
Hi Matt,

Yes, those two are both valid possibilities, but do not show up as answer choices in Question 12. The key inference in this game are that L and S must both be held on Tuesday. So Question 12 forces L into Monday and Tuesday, and S into Tuesday and Wednesday. This immediately eliminates answer choices (A), (B), (D) and (E), leaving us only with (C).
 mattnj
  • Posts: 13
  • Joined: Aug 19, 2017
|
#41676
Thanks James.. With another review, I realized this is a "could be" question, so choice C is correct. If it was a "must be" question the 2 additional combinations I mentioned below would have made C incorrect.

Thanks again!

Get the most out of your LSAT Prep Plus subscription.

Analyze and track your performance with our Testing and Analytics Package.